Time Left - 20:00 mins

CR- STATEMENTS AND ARGUMENTS || Verbal Ability || CAT 2021 || 3 June

Attempt now to get your rank among 239 students!

Question 1

The question given below consists of a statement, followed by three arguments numbered I, II and III. You have to decide which of the given arguments is/are ‘strong’ arguments and which is/are ‘weak’ arguments and accordingly choose your answer from the alternatives given below each question.
Statement:
A shortage of bank branches and ATMs across India’s hinterland is holding back the Prime Minister's financial inclusion efforts and risks angering rural voters ahead of elections next year. After taking office, the PM set an ambitious target to open a bank account for every household to ensure welfare funds flow directly to India’s poor while improving access to credit and insurance programs. He pushed policies that helped bring 310 million people into the formal banking system in just four years, according to the World Bank.

Based on the arguments stated below and the information stated above, which of the following arguments state the reason for the problem, ‘But many of India’s villages still lack bank branches or ATMs to help service new customers, while the pace of building new financial infrastructure has actually slowed’.

Arguments:

I. Because the central government effectively forced poor citizens into the banking system by linking some welfare benefits to bank accounts, villagers have ended up stuck in long queues and struggling with ATMs that often run out of cash or break down.
II. With an election due next year, the mismatch between the government’s policies and the rural banking system is generating frustration among a key slice of India’s electorate.
III. The banking system struggled to keep up, while some gains proved temporary. Nearly half of the Indian bank accounts were inactive in 2007, meaning they weren’t used at all in the last 12 months

Question 2

Direction: In each question below is given a statement followed by three assumptions numbered I, II and III. An assumption is something supposed or taken for granted. You have to consider the following assumption and decide which of the assumption is implicit in the statement.
Statement: A moment of silence, please, for the world’s cheapest car, which has all but died in India. It was almost 10 years old. The Nano’s death was confirmed by production numbers: Tata Motors Ltd. produced 1 unit in June, down from 275 in the same month last year. Exports were zero, versus 25 in June 2017.

The company acknowledged that the car in its ‘present form cannot continue beyond 2019’. Why?

Arguments:
I. The expiry of the ‘people’s car’, as Tata Motors branded it in 2008, holds lessons for automakers hoping to make it in India: While consumers may be value-conscious, cutting costs to the bone in pursuit of a gimmicky claim to fame is no use if the end result is a second-rate vehicle with a tendency to catch fire.
II. The Nano’s failure to sell stands in stark contrast to the rest of the Indian car market. From motorbikes to cars and trucks, growth in every segment is picking up.
III. This is a market where electric vehicles are non-existent and the chatter over fancy ideas such as future mobility and autonomous vehicles, which pervades the rest of the auto world, is muted.

Question 3

Direction: In the question below, a statement is given followed by three arguments. Choose the most appropriate option depending on which argument strengthens the given statement.
Statement:
Should the government ban the export of all types of foodgrains for the next year to tide over the unpredicted drought situation in the country?

Arguments: 
I. Yes, there is no other way to provide food to its citizen during the year.
II. No, the government does not have its jurisdiction over private exporters for banning exporters.
III. Yes, the government should not allow the exporters to export foodgrains and procure all the foodgrains held by such exporters and make it available for home consumption.

Question 4

The question given below consists of a statement, followed by three arguments numbered I, II and III. You have to decide which of the arguments is/are ‘strong’ arguments and which is/are ‘weak’ arguments and accordingly choose your answer from the alternatives given below each question.
Statement:
World’s largest beer maker Heineken NV, the majority partner of United Breweries with Vijay Mallya, is understood to have sought legal opinion over its right to appoint a chairman at the Indian company. Heineken and some of its advisers believe that the shareholder agreement between Mallya and the beer giant has become null and void after India’s Enforcement Directorate attached his shares as part of its legal action against the liquor baron.

Which among the following arguments support the above statement in the best possible manner?

Arguments:
I. UBL has stopped sharing confidential information with Mallya and has said that he is no longer privy to any strategic developments.
II. The board of UBL, India’s biggest beer company, had asked Mallya to either step down or appoint a nominee after the Securities and Exchange Board of India barred wilful defaulters from holding key board positions last year.
III. The company is functioning well and operations are in good shape but it is not good corporate governance to have an acting chairman for so long.

Question 5

Direction: The question given below consists of a statement, followed by two arguments numbered I and II. You have to decide which of the arguments is a 'strong' argument and which is a 'weak' argument.
Statement:
Should parents be involved in arranging their child’s marriage?

Arguments:
I. Yes, because this is what has been happening for generations.
II. No, because every child should be given the opportunity to live on their own terms and to make choices on their own.

Question 6

Directions: Read the following statements carefully and answer the questions which follow.
According to a recent government directive, all bank branches in rural areas should be computerised.
Which of the following statements would weaken the government's argument?

Question 7

There has been a rapid increase in the demolition of some of the world's most precious sites in and around the Syrian-Jordanian border with the rapid expansion of extremist forces. The ancient city of Palmyra has recently been demolished by a radical terrorist group in its attempt to dominate the Syrian landscape. This proves their attempt to wipe out the ancient history of the region dating back years before the advent of Islam as the major religion.

Which of the following, if true, would most strengthen the author's conclusion regarding the attempts of the radical terrorist group?

Question 8

Online shopping is an underexplored area of merchandising musicology. A new study commissioned by eBay aims to correct that. Some 1900 participants were asked to simulate online shopping by listening to different sounds. The noise of road-works and crying babies soured the shoppers' views of the products on offer. Sounds of birds encouraged sales of barbecues but not blenders or board games. Sounds associated with quality and luxury seemed to be hazardous for shoppers' wallets. The study found that classical music and restaurant buzz caused them to overestimate the quality of goods on offer and pay more than they should.
Which of the following, if true, most strengthens the argument above?

Question 9

Women who have a poor diet during their pregnancies will have children who are more susceptible to age related diseases than those who have a healthier diet, scientists say. The warning comes after the research found that rats that had poor nutrition during pregnancy gave birth to young with a high risk of type 2 diabetes which is an illness that typically strikes in the middle age.

Which of the following, if true, most strengthens the conclusion above?

Question 10

Direction: In each question below is given a statement followed by two assumptions numbered I and II. You have to consider the statement and the following assumptions and decide which of the assumptions is implicit in the statement.
Statement:
The concession in rail fares for journeying to hill stations has been cancelled because it is not needed by people who can spend their holidays there.

Assumptions:
I. Railways should give concession only to the needy.
II. Railways should not encourage people to spend their holidays at hill stations.
  • 239 attempts
  • 1 upvote
  • 2 comments
Jun 24CAT & MBA